Is \phi'(x)=\phi(x') a derivable identity under Lorentz transformations?

In summary, a Lorentz transformation is a linear transformation that preserves the relativistic line element. It is defined as x^\mu \rightarrow x'^\mu = L^\mu{}_\nu, where x'^\mu = (L^{-1})^\mu{}_\nu x^\nu. The equation (*) is derived from this definition.
  • #1
latentcorpse
1,444
0
a)So I'm reading over my notes and they say that under the Lorentz transformation L, [itex]\phi \rightarrow \phi'[/itex] where [itex]\phi'(x)=\phi(x')[/itex] where [itex]x'^\mu = (L^{-1})^\mu{}_\nu x^\nu[/itex]

I don't really understand why this is true.

Why is it not just [itex]\phi'(x)= L \phi(x)[/itex]
Clearly this fails because the LHS is a scalar and the RHS should have indices on the L and so it won't be a balanced tensor equation but to my mind this is the right "form" that the equation should have.

Can anyone explain this to me?b)However, earlier in my notes when Lorentz transformations were introduced it says
A Lorentz transformation is a linear transformation on space and time

[itex]x^\mu \rightarrow x'^\mu = L^\mu{}_\nu[/itex]
which preserves the relativistic line element [itex]ds^2[/itex]
and so
[itex]L^\mu{}_\sigma L^\nu{}_\tau g^{\sigma \tau} = g^{\mu \nu}[/itex] (*)

Why does it define the transformation as [itex]x^\mu \rightarrow x'^\mu = L^\mu{}_\nu[/itex] here and as [itex]x'^\mu = (L^{-1})^\mu{}_\nu x^\nu[/itex] in the other part of my notes (as I mentioned in part a) of my post) - are these equivalent? If so, how?

And how do we derive the equation (*)Thanks a lot.
 
Last edited:
Physics news on Phys.org
  • #2
latentcorpse said:
a)So I'm reading over my notes and they say that under the Lorentz transformation L, [itex]\phi \rightarrow \phi'[/itex] where [itex]\phi'(x)=\phi(x')[/itex] where [itex]x'^\mu = (L^{-1})^\mu{}_\nu x^\nu[/itex]

I don't really understand why this is true.

Why is it not just [itex]\phi'(x)= L \phi(x)[/itex]
Clearly this fails because the LHS is a scalar and the RHS should have indices on the L and so it won't be a balanced tensor equation but to my mind this is the right "form" that the equation should have.

Can anyone explain this to me?

I think your just forgetting that [itex]\phi[/itex] is a scalar field. Scalars don't transform. That is the definition of a scalar. We don't multiply by a lambda because, by definition, our scalar quantity is invariant!

Think of the definition of tensors (Don't think of these tensors as fields yet.) by their transformation properties:

Rank 0 (Scalar): [itex]\phi\rightarrow\phi[/itex]

Rank1(Vector):[itex]x^{\mu}\rightarrow\Lambda^{\nu}_{\mu}x^{\mu}[/itex]

Rank2:[itex]A_{\mu\nu}\rightarrow\Lambda^{\mu}_{\rho}\Lambda^{\nu}_{\sigma}A_{\mu\nu}[/itex]

Again, we don't have a Lambda matrix multiplying [itex]\phi[/itex] because it is defined as a scalar. This is just what a scalar is! However,in our case, our scalar is a field, defined at all points in space. This means it depends on spatial coordinates, which transform like vectors. Thus, we need to multiply the coordinates by lambda^-1 to transform them correctly.

I hope this helps explain why there is a lambda "inside" but not "outside."
b)However, earlier in my notes when Lorentz transformations were introduced it says
A Lorentz transformation is a linear transformation on space and time

[itex]x^\mu \rightarrow x'^\mu = L^\mu{}_\nu[/itex]
which preserves the relativistic line element [itex]ds^2[/itex]
and so
[itex]L^\mu{}_\sigma L^\nu{}_\tau g^{\sigma \tau} = g^{\mu \nu}[/itex] (*)

Why does it define the transformation as [itex]x^\mu \rightarrow x'^\mu = L^\mu{}_\nu[/itex] here and as [itex]x'^\mu = (L^{-1})^\mu{}_\nu x^\nu[/itex] in the other part of my notes (as I mentioned in part a) of my post) - are these equivalent? If so, how?

And how do we derive the equation (*)Thanks a lot.

I'm new to field theory myself, so I'll think about this and hopefully get back to you. If not, I'll inform another homework helper about your question.
 
Last edited:
  • #3
G01 said:
I think your just forgetting that [itex]\phi[/itex] is a scalar field. Scalars don't transform. That is the definition of a scalar. We don't multiply by a lambda because, by definition, our scalar quantity is invariant!

Think of the definition of tensors (Don't think of these tensors as fields yet.) by their transformation properties:

Rank 0 (Scalar): [itex]\phi\rightarrow\phi[/itex]

Rank1(Vector):[itex]x^{\mu}\rightarrow\Lambda^{\nu}_{\mu}x^{\mu}[/itex]

Rank2:[itex]A_{\mu\nu}\rightarrow\Lambda^{\mu}_{\rho}\Lambda^{\nu}_{\sigma}A_{\mu\nu}[/itex]

Again, we won't have a Lambda matrix multiplying [itex]\phi[/itex] because it is defined as a scalar. This is just what a scalar is!


However,in our case, our scalar is a field, defined at all points in space. This means it depends on spatial coordinates, which transform like vectors. Thus, we need to multiply the coordinates by lambda^-1 to transform them correctly.

I hope this helps explain why there is a lambda "inside" but not "outside

ok. but shouldn't it then be [itex]\phi'(x)=\phi(x)[/itex] as the transformation law?

or is the following correct:

consider [itex]\phi'(x)[/itex]
by defn this is equal to [itex]\phi(x')[/itex] (is this actually just a defn i should accept?)
and then we do the lorentz transformation of the spatial coords to get the desired result?

thanks again!
 
  • #4
latentcorpse said:
ok. but shouldn't it then be [itex]\phi'(x)=\phi(x)[/itex] as the transformation law?
Remember that you are still moving from one frame to another. Thus, we need to transform to the coordinates of the new frame. In the above statement, you are essentially say the following:

"Our field is a scalar. Therefore, The field value in frame 1 written in terms of frame 1's coordinates is equal to the field value in frame 2, also written in terms of frame 1's corrdinates."

This is not true! Obviously when we are working in a new frame, we want to work with those new coordinates. Otherwise we will be comparing field values for different spatial locations, which obviously don't have to be equal. So what we really want to say is the following:

"Our field is a scalar. Therefore, the field value in frame 1 written in terms of frame 1's coordinates is equal to the field value in frame 2, written in terms of frame 2's coordinates."

Or in math:

[itex]
\bar{\phi}(\bar{x})=\phi(\bar{x})=\phi(\Lambda^{-1}x)
[/itex]

Again, I'll get back to you on why we use lambda inverse.
 
Last edited:
  • #5
G01 said:
Remember that you are still moving from one frame to another. Thus, we need to transform to the coordinates of the new frame. In the above statement, you are essentially say the following:

"Our field is a scalar. Therefore, The field value in frame 1 written in terms of frame 1's coordinates is equal to the field value in frame 2, also written in terms of frame 1's corrdinates.'

This is not true! Obviously when we are working in a new frame, we want to work with those new coordinates. Otherwise we will be comparing field values for different spatial positions, which obviously don't have to be equal. So what we really want to say is the following:

"Our field is a scalar. Therefore, the field value in frame 1 written in terms of frame 1's coordinates is equal to the field in frame 2, written in terms of frame 2's corrdinates."

Or in math:

[itex]
\bar{\phi}(x)=\phi(\bar{x})=\phi(\Lambda^{-1}x)
[/itex]

surely you mean
[itex]\phi'(x)=\phi(x') = \phi( \Lambda^{-1} x)[/itex]

ok. but i don't get why we don't write

[itex]\phi'(x') = \phi(x)[/itex]

i guess I'm just confused by us having primes on both sides of the equation rather than having them all on one side - surely this woould make more sense if we took a prime to mean frame 1 then what I've written above would say field in frame 1 in frame 1 coords equals field in frame 2 in frame 2 coords but [itex]\phi'(x)=\phi(x')[/itex] means field in frame 1 in frmae 2 coords equals field in frame 2 in frame 1 coords...i'm a bit confused still.

sorry!
 
  • #6
latentcorpse said:
surely you mean
[itex]\phi'(x)=\phi(x') = \phi( \Lambda^{-1} x)[/itex]

Yes I had a typo, but what I meant was:

[itex]\phi'(x')=\phi(x') = \phi( \Lambda^{-1} x)[/itex]

Does this help? Sorry for the mistake. (I'm not much farther into this stuff than you. I'm afraid we may be an example of the blind leading the blind!)

Also, the appearance of the inverse lambda in the field argument has to do with the difference between active and passive transforms. Check out the "Lorentz invariance" section of these lecture notes from David Tong:

http://www.damtp.cam.ac.uk/user/tong/qft/one.pdf

The whole set of notes is at:

http://www.damtp.cam.ac.uk/user/tong/qft.html

I found these notes quite helpful, especially for the discussion of classical field theory and Lorentz invariance.
 
  • #7
G01 said:
Yes I had a typo, but what I meant was:

[itex]\phi'(x')=\phi(x') = \phi( \Lambda^{-1} x)[/itex]

Does this help? Sorry for the mistake. (I'm not much farther into this stuff than you. I'm afraid we may be an example of the blind leading the blind!)

Also, the appearance of the inverse lambda in the field argument has to do with the difference between active and passive transforms. Check out the "Lorentz invariance" section of these lecture notes from David Tong:

http://www.damtp.cam.ac.uk/user/tong/qft/one.pdf

The whole set of notes is at:

http://www.damtp.cam.ac.uk/user/tong/qft.html

I found these notes quite helpful, especially for the discussion of classical field theory and Lorentz invariance.

yeah. thanks for that. i now understand the inverse lambda term.

still not sure why [itex]\phi'(x)=\phi(x')[/itex]? is this just a definition?
 
  • #8
ok. part b) actually just follows quite neatly as a consequence of special relativity. The relativistic line element ds^2 must be invariant under Lorentz transformations and if you write it out it all works out.

So I just need clarification of whether or not [itex]\phi'(x)=\phi(x')[/itex] is an identity that I should just accept or whether it is in fact derivable?
 

What are Lorentz Transformations?

Lorentz Transformations are a set of mathematical equations that describe the relationship between space and time in the theory of special relativity. They were developed by Dutch physicist Hendrik Lorentz in the late 19th century.

Why are Lorentz Transformations important?

Lorentz Transformations are important because they allow us to understand how measurements of space and time change for observers in different reference frames. This is crucial in the theory of special relativity, which states that the laws of physics are the same for all observers in uniform motion.

What is the difference between Galilean Transformations and Lorentz Transformations?

Galilean Transformations were used to describe the relationship between space and time in classical mechanics, while Lorentz Transformations were developed to account for the effects of relativity in high-speed scenarios. Unlike Galilean Transformations, Lorentz Transformations take into account the constant speed of light and the relativity of simultaneity.

How do Lorentz Transformations affect our understanding of time and space?

Lorentz Transformations have shown that time and space are not absolute, but rather relative to the observer's reference frame. This means that measurements of time and space can differ for observers in different reference frames, and the concept of simultaneity is also relative.

What are some practical applications of Lorentz Transformations?

Lorentz Transformations have been applied in various fields, such as physics, engineering, and astronomy. They are used in the development of technologies such as GPS systems, particle accelerators, and nuclear reactors. They have also been crucial in the development of the theory of relativity and our understanding of the universe.

Similar threads

  • Advanced Physics Homework Help
Replies
1
Views
321
  • Advanced Physics Homework Help
Replies
8
Views
731
Replies
5
Views
2K
  • Advanced Physics Homework Help
Replies
30
Views
5K
  • Advanced Physics Homework Help
Replies
5
Views
2K
  • Advanced Physics Homework Help
Replies
1
Views
1K
  • Advanced Physics Homework Help
Replies
0
Views
456
  • Advanced Physics Homework Help
Replies
2
Views
2K
  • Advanced Physics Homework Help
Replies
4
Views
1K
Replies
5
Views
393
Back
Top